LSAT and Law School Admissions Forum

Get expert LSAT preparation and law school admissions advice from PowerScore Test Preparation.

User avatar
 Dave Killoran
PowerScore Staff
  • PowerScore Staff
  • Posts: 5850
  • Joined: Mar 25, 2011
|
#27423
Complete Question Explanation
(The complete setup for this game can be found here: lsat/viewtopic.php?t=11593)

The correct answer choice is (E)

This is the most difficult question of the game. Each of the answers seem vague—in direct contrast to the final diagram, which is quite specific. The correct answer choice, (E), uses the Overlap Principle.

In answer choice (E) the Overlap Principle applies to the lectures and operations on Monday, Tuesday and Wednesday. Two operations and two lectures must be assigned within this three day period and consequently there must be an overlap between the two groups. Consider all the possible permutations of the lectures and operations for those days:
Feb 93__M12_game#2_L11_explanations_game#2_#10_diagram_1.png
In each case at least one lecture is given on a day the doctor operates, and sometimes it occurs twice.
 destaat
  • Posts: 17
  • Joined: Aug 07, 2012
|
#5641
My complete diagram tells me that answer choice (c) is the only thing that must happen. Can someone explain why this is wrong and why answer choice (e) is the best answer?

Please, I would really appreciate the input. Thank you.

-A.T.
User avatar
 Dave Killoran
PowerScore Staff
  • PowerScore Staff
  • Posts: 5850
  • Joined: Mar 25, 2011
|
#5680
Hi A.T.,

This is the most difficult question of the game. Each of the answers seem vague—in direct contrast to the final diagram, which is quite specific. Without seeing your final diagram, it is hard to know exactly where you went wrong. If you have the Logic Games Bible, that game appears on page 167 with the explanations immediately thereafter. That game is also used in our courses (with explanations) and explained in our LSAT Setups Encyclopedia, if you were in a course or have that book.

Answer choice (C) can occur, but it does not have to occur. For example, on Tuesday, Yamata could Research in the morning and Lecture in the afternoon, and thus Researching and Treating would not occur on the same day

The correct answer choice, (E), uses the Overlap Principle. In answer choice (E) the Overlap Principle applies to the lectures and operations on Monday, Tuesday and Wednesday. Two operations and two lectures must be assigned within this three day period and consequently there must be an overlap between the two groups.

Please let me know if that helps. Thanks!
 destaat
  • Posts: 17
  • Joined: Aug 07, 2012
|
#5727
This does. Looking back I made a mistake on my diagram. For some reason i put down R as having must worked on Saturday morning. And I didn't realize that this game was in the logic games bible. Thank you Dave for the solid explanation.
 Annah
  • Posts: 16
  • Joined: Jul 21, 2013
|
#11097
Dear Dave,

My first question is: How exactly do you recognize when a question requires the application of the overlapping principle? Are there any key words or phrases to look out for? Do all questions that require one to pick out variables that are repeated in two different variable groups lead to the application of the principle?

Secondly, in Question no.10, Game#4, February 1993, Pg. 167 of the Logic Games Bible (page 223 of the new LGB) how do you determine which variables the overlap principle applies to when all the variables overlap to some degree on different days? Do you pick the variable sets that yield the highest number of combinations in terms of overlapping?
For instance in part (A) of Question 10, there are at least 3 instances where the doctor treats patients both in the morning and afternoon:

Morning: T,O,O,O,R
Afternoon: T,L,L,T,T

Morning: R,O,O,O,T
Afternoon: T,L,L,T,T

Morning: R,O,O,O,T
Afternoon: L,L,T,T,T

For part (C) the following six combinations can be yielded where the two variables 'R' and 'T' are overlapping/repeated on the same day:

Morning: T,O,O,O,R
Afternoon: L,L,T,T,T

O,T,O,O,R
L,L,T,T,T

T,O,O,O,R
T,L,L,T,T

O,T,O,O,R
T,L,L,T,T

R,O,O,O,T
T,L,L,T,T

R,O,O,O,T
T,L,L,T,T

Part (E) yields the largest number of combinations where the variables 'L' and 'O' overlap.
In my diagram it yields 8 different combinations.

Is it true that the answer that 'must' be true in a question such as this is one with the highest number of combinations? (Option E here). If that is the case, it takes a significant amount of time to create these combinations and I would like to know whether there's is a quicker solution for arriving at the correct answer without having to resort to such tedious methods.

I feel as if I am approaching the question from an inaccurate perspective. Please correct me if that is the case. The explanation in the answer provided in the book was too brief for what seems like a rather complicated question.

Thank you.
User avatar
 Dave Killoran
PowerScore Staff
  • PowerScore Staff
  • Posts: 5850
  • Joined: Mar 25, 2011
|
#11107
Hi Anna,

Good to hear from you. As usual, you've asked some great questions :-D

In answer to your first question, I search for and recognize the presence of overlap when it occurs as part of my routine examination of each game. Clearly, a lot of games do not have overlap elements, so I'm not looking for them there. What, then, triggers the thought that it might be an issue? The following:
  • In Linear Games, the presence of multiple variable sets, or repeating variable sets (such as in this game). That means that Basic Linear games won't have overlap, but Advanced Linear games (again, like the one we are discussing) do.

    In Grouping Games, the presence of variables with multiple characteristics (what I refer to as Subdivided). For example, a game might specify that a panel of five doctors is being chosen from various hospitals, and then add that each of those doctors has a specialty (neurology, etc). That could lead to overlap scenarios such as, "At least one of the doctors from Fairview is a cardiologist."
In the questions, there is not a particular type of question (Must, Could, etc) that generates an overlap result, but the language of the answers will use some type of phrasing that reflects that direction, like "at least" or "both." the problem is, that language shows up in other types of questions too. the key is that you are at the mercy of the test makers: overlap only comes into play if they ask about it, or if a key inference trades on that idea.

In answer to your second question about #10, I looked at this question in a different way. Instead of it being two sets of 5 variables each, I looked at it as two groups: one of three days and one of two days. The two day group was unlikely to be the source of the answer because both afternoons are T. In the three-day group, we know now what occurs.

In the second part of the the second question, you've made an interesting analysis of the number of times each event occurs. I feel like this is useful in a post-game, find-all-the angles type of analysis, but as you note, it's not the fastest or most efficient solution during the game. However, it is the case that in a Must Be True question, the "one with the highest number of combinations" must be the correct answer. This occurs, as I'm sure you already suspect, because that event must occur in every single solution, meaning it maxes out in terms of event numbers. Since in many games you can't take the time to write out all the solutions, the two ways to solve this question are to 1. seek solutions that do not include the event described in the answer choice and 2. reduce the groups in the manner I describe above (2/3 vs 5/5). That second solution becomes a lot easier if you add overlap scenario recognition into your initial game analysis, and then keep it in reserve in case a question comes up about it. Regardless, this is as I note in the book, the hardest question of the game.

As an aside, I'm rather interested in how you are performing overall on the LG section (and the test as a whole). If you don't feel comfortable sharing that here, please send me a PM. You have an insightful way of seeing the test, and I'd like to talk to you more about it.

Thanks!
 Annah
  • Posts: 16
  • Joined: Jul 21, 2013
|
#11444
Dave,

Thank you very much for this explanation and please check your inbox :)
 ddion8206
  • Posts: 10
  • Joined: Nov 17, 2015
|
#22269
Can someone explain to me question 10? 10 Confused me because its a must be true but the correct answer was kinda like a could be true.

Thank you.
 Emily Haney-Caron
PowerScore Staff
  • PowerScore Staff
  • Posts: 577
  • Joined: Jan 12, 2012
|
#22270
Hi ddion,

For number 10, it is actually a must be true, but only if you make all the inferences. She will operate on either Monday or Tuesday AND on Wednesday in the morning; in the afternoon, she will lecture either Monday/Tuesday or Tuesday/Wednesday. If she operates Monday-Wednesday, we know she'll have to lecture either on Monday or Wednesday, so she'll do both on one day (we just don't know which day). If she operates Tuesday-Wednesday, we know she will lecture on Tuesday, so we know she does both that day. Either way, she will operate and lecture on a single day.
 k100
  • Posts: 10
  • Joined: Nov 21, 2019
|
#76744
Hi,

Though I arrived at the right answer, I thought answer choice (E) should instead state "the doctor lectures on at least one of the days..." because she could both lecture and operate on Tuesday and Wednesday. Since this is a must be true question, isn't (E)'s wording too strong in this case? It seems to say that the doctor lectures on [exactly] one of the days on which she operates.

Thank you!

Get the most out of your LSAT Prep Plus subscription.

Analyze and track your performance with our Testing and Analytics Package.